Đến nội dung

lahantaithe99

lahantaithe99

Đăng ký: 18-12-2013
Offline Đăng nhập: 30-07-2019 - 11:54
*****

#563152 $A=8(a+b+c)+5(\frac{1}{a}+\frac{1...

Gửi bởi lahantaithe99 trong 02-06-2015 - 23:57

  Cho các số thực dương $a,b,c> 0$ thỏa mãn $a^2+b^2+c^2=3$. Tìm GTNN của:

 

           $A=8(a+b+c)+5(\frac{1}{a}+\frac{1}{b}+\frac{1}{c})$

 

Cách khác

 

Áp dụng BĐT phụ là $(a+b+c)^5\geq 81abc(a^2+b^2+c^2)$ thì theo điều kiện bài toán

 

$(a+b+c)^5\geq 243abc\geq 243(abc)^{\frac{5}{2}}\Rightarrow \left ( \frac{a+b+c}{\sqrt{abc}} \right )^5\geq 243\rightarrow \frac{a+b+c}{\sqrt{abc}}\geq 3$

 

Theo $AM-GM$: $A\geq 13\sqrt[13]{\frac{(a+b+c)^{13}}{(abc)^5}}=13\sqrt[13]{\frac{(a+b+c)^5}{abc}.\left ( \frac{a+b+c}{\sqrt{abc}} \right )^8}\geq 13\sqrt[13]{243.3^8}=39$

 

Vậy min $A=39$




#563147 Nếu $p^{2l-1}m(mn+1)^2+m^2$ là một số chính phương thì...

Gửi bởi lahantaithe99 trong 02-06-2015 - 23:22

Phương trình $m(p^{2l-1}(mn+1)^2+m)=a^2(a\in \mathbb{N}^*)$

 

TH1 :$gcd(m,p^{2l-1}(mn+1)^2+m)=1$ thì hiển nhiên $m$ là số chính phương

 

TH2 :$gcd(m,p^{2l-1}(mn+1)^2+m)=d$ ( $d>1$) thì $d|p^{2l-1}\rightarrow d=p^k$ ( $k\leq 2l-1$)

Lúc đó đặt $m=p^k.t$. Dễ thấy $(t,p)=1$

Phương trình trở thành $t(p^{2l-1-k}(mn+1)^2+t)=u^2$ ( $up^k=a$) có $(t,p^{2l-1-k}(mn+1)^2+t)=1$ nên $\left\{\begin{matrix} t=x^2 & \\ p^{2l-1-k}(mn+1)^2+t=y^2 & \end{matrix}\right.$

Để kết thúc bài toán thì ta cần chứng minh $k$ chẵn. Giả sử $k$ lẻ thì $2l-1-k$ chẵn . Đặt $2l-1-k=2u$ thì

$[p^u(mn+1)^2]+x^2=y^2$ là số chính phương

 

Mà $[p^u(mn+1)]^2<[p^u(mn+1)]^2+x^2=[p^u(mn+1)]^2+\frac{m}{p^k}<[p^u(mn+1)+1]^2$ nên điều trên vô lí

Vậy $k$ chẵn. Do đó $m=p^kx^2$ là số chính phương




#563055 Giải phương trình 2 ẩn nguyên dương : $x^{y^{x}}=y^...

Gửi bởi lahantaithe99 trong 02-06-2015 - 17:03

Có $x=y$ là nghiệm của phương trình

Nếu $x\neq y$. Do vai trò $x,y$ như nhau nên giả sử $x>y$. Khi đó $\left ( \frac{x}{y} \right )^{y^x}=y^{x^y-y^x}>1\Rightarrow x^y>y^x$ và $t=\frac{x}{y}\in\mathbb{N},t>1$

 

Đặt $x=p_1^{m_1}.....p_k^{m_k}$ thì  $y=p_1^{n_1}...p_k^{n_k}$, gcd$(p_i,p_j)=1\forall i\neq j$.

Thu được $\frac{m_i}{n_i}=\frac{x^y}{y^x}=\frac{t^y}{y^{y(t-1)}}>1\Leftrightarrow t^y>y^{y(t-1)}\rightarrow t>y^{t-1}$

Mà bằng quy nạp ta dễ dàng chứng minh với $y,t\in\mathbb{N^*}$ thì  $y^{t-1}\geq (y-1)(t-1)+1\geq t$

Dấu $=$ xảy ra khi $t-1=0$ ( loại ) hoặc $t-1=1\rightarrow t=2\rightarrow x=y=1$ ( vô lí vì $x\neq y$ )

Vậy $x=y\in\mathbb{N^*}$ là nghiệm của phương trình




#562888 CMR $m=p$ nếu $\frac{x^p+y^p}{2}=...

Gửi bởi lahantaithe99 trong 01-06-2015 - 19:22

Từ giả thiết dễ có : $m\geq p$ và $2|x+y$. Đặt $x+y=2^n.t$ ( $(2,t)=1,n>1$)

TH1: $t=1$

 +) $p=2$ thì $x^2+y^2=\left ( \frac{x+y}{2} \right )^m$. Từ đk $(x,y)$ nguyên dương không cùng bằng $1$ dễ dàng chặn được $m<3$ nên $m=p=2$

 +) $p$ lẻ: Ta thu được $x^{p-1}-x^{p-2}y........-xy^{p-2}+y^{p-1}=2^{(m-1)(n-1)}$ chia hết cho $4$ $(1)$

-Nếu $x,y$ lẻ thì do $4|2^{n}=x+y$ nên ta giả sử $x\equiv 1$ (mod $4$) và $y\equiv -1$ (mod $4$)

Thế thì $x^{p-1}+....+y^{p-1}\equiv p\not\equiv 0$  (mod $4$) : mâu thuẫn với $(1)$

-Nếu $x,y$ chẵn chỉ cần đặt $x=2^uw, y=2^sv$ ( $w,v$ lẻ). Biến đổi và lí luận tương tự như trên cũng suy ra điều vô lý

TH2: $t\neq 1$. Khi đó tồn tại $q|t$ ( $q\in\mathbb{P}$ lẻ )

+) Nếu $q|x+y$ và $q$ không là ước của  $x,y$ thì áp dụng bổ đề LTE ta có

$v_q(x^p+y^p)=v_q[(x+y)^m]=v_q(x+y)+v_q(p)=mv_q(x+y)$. Điều này dẫn đến $m=2\rightarrow p=2$ (loại vì $p$ lẻ)

+) Nếu $q|x+y$ và $q|x,q|y$. Đặt $x=q^a.l,y=q^b.c$ ( $(l,q)=(c,q)=1$).

   -$a=b$: lí luận như trên

   -$a\neq b$. Giả sử $a>b$. Khi đó: $2^{m-1}q^{bp}(q^{ap-bp}.l^p+c^p)=q^{bm}(q^{a-b}l+c)^m$. Điều này cho ta $m=p$

 

 

 




#561437 $1+2^x+2^{2x+1}=y^2$

Gửi bởi lahantaithe99 trong 25-05-2015 - 00:31

Tìm các số nguyên x,y thỏa mãn :

                    $1+2^x+2^{2x+1}=y^2$

 

Phương trình tương đương: $(1+2^{x-1})^2+7.2^{2x-2}=y^2$ $\Leftrightarrow 7.2^{2x-2}=(y-1-2^{x-1})(y+1+2^{x-1})$

 

Đến đây xét TH kết hợp với điều kiện $y$ lẻ là được




#560099 $x^{7}+y^{7}=1998^{z}$

Gửi bởi lahantaithe99 trong 18-05-2015 - 07:13

Đặt $d$ là ước chung lớn nhất của $x,y$ thì $x=da, y=db$ sao cho $(a,b)=1$

 

Khi đó $d^7( a^7+b^7)=1998^z$ $(*)$

 

Nếu $a=b=1$ thì dễ thấy vô lý nên $(a,b)\neq (1,1)$

 

Khi đó vì $(a,b)=1$ nên áp dụng định lý Zsigmondy's thì luôn tồn tại số nguyên tố $p$ thỏa mãn $p|a^7+b^7$ và $p$ không là ước của $a+b$

 

Từ $(*)$ suy ra $p=2,3,37$

 

+) Nếu $p=2$ thì vô lí

 

+) Nếu $p=3$ thì do $(a,b)=1$ và $a+b$ không chia hết cho $3$ nên $a\equiv b\equiv -1,1$ ( mod $3$)

 

Khi đó $a^6-a^5b+.....-ab^5+b^6\equiv 1$ (mod $3$) nên $a^7+b^7$ không chia hết cho $3$ (loại )

 

+) Nếu $p=37$ . Do $(a,b)=1$ nên $(a,37)=(b,37)=1$

 

$a^7\equiv -b^7$ (mod $37$) $\rightarrow a^{35}\equiv -b^{35}$ (mod $37$ ) $(1)$

 

Mặt khác theo định lí Fermat thì $a^{36}\equiv b^{36}\equiv 1$ (mod $37$) $(2)$

 

Từ $(1)$ và $(2)$ dễ dàng suy ra $37|a+b$ ( vô lí)

 

Do đó không tồn tại số $p$ thỏa mãn nên phương trình vô nghiệm

 

 




#559978 Cho $a,b,c>0 ; \sum a^2=3 . CM 5(a+b+c)+\frac{3}...

Gửi bởi lahantaithe99 trong 17-05-2015 - 15:19

Cho $a,b,c >0$ và $a^2+b^2+c^2=3$ . Chứng minh $A=5(a+b+c)+\frac{3}{abc}\geq 18$

BĐT phụ sau đây rất hữu hiệu để chứng minh bài toán:

 

Cho $a,b,c>0$ thì có $(a+b+c)^5\geq 81abc(a^2+b^2+c^2)$

 

Như vậy, đặt $a+b+c=3t$ ( $t>0$) thì áp dụng BĐT phụ trên và $AM-GM$

 

 $A\geq 15t+\frac{3}{t^5}=3t+3t+3t+3t+3t+\frac{3}{t^5}\geq 6\sqrt[6]{3^6}=18$ ( đpcm)

___________________________________________

 

P/s: Để chứng minh BĐT phụ, ta có thể dùng pqr hoặc cách quen thuộc ở

http://diendantoanho...c5geq-81a2b2c2/




#559065 Giải phương trình nghiệm nguyên $2^x+1=x^2y$

Gửi bởi lahantaithe99 trong 13-05-2015 - 10:27

Giải phương trình nghiệm nguyên $2^x+1=x^2y$

 

Từ điều kiện đề bài ta có $x^2|2^{2x}-1$. Gọi $p$ là ước nguyên tố nhỏ nhất của $x$ thì $p|2^{2x}-1$

 

Áp dụng định lý Fermat nhỏ thì $p|2^{p-1}-1$. Gọi $t$ là số nhỏ nhất sao cho $p|2^t-1$

 

Khi đó ta có ngay $t|2x$ và $t|p-1\rightarrow t<p$

 

Nếu $t$ có chứa ước nguyên tố lẻ nào đó ( hoặc $t$ nguyên tố) thì ước đó sẽ là ước của $x$ và nhỏ hơn $p$ ( vô lý do $p$ nhỏ nhất)

 

Do đó $t=2$ . Suy ra $p=3$

 

Đặt $x=3^k.v$ ( $(3,v)=1$) thì $3^{2k}|2^{3^kv}+1$

 

Khi đó theo bổ đề LTE: $v_3(2^{3^kv}+1)=1+k\geq 2k\Rightarrow k=1\rightarrow v^2|8^v+1$

 

+)  Nếu $v=1$ thì $n=3$ ( thỏa mãn )

 

+) Nếu $v\geq 2$ .Lại gọi $h$ là ước nguyên tố nhỏ nhất của $v$  và tương tự như trên, ta thu đc $h=7$

 

Do đó $7|8^v+1$ . Vô lý vì $8^v+1\equiv 2$ (mod $7$ ) 




#558781 CMR: $\sum \sqrt{\frac{a^{2}}...

Gửi bởi lahantaithe99 trong 11-05-2015 - 20:13

 

Cho a,b,c > 0 và $a^{2}+b^{2}+c^{2}=3$. CMR:

$\sum \sqrt{\frac{a^{2}}{a^{2}+b+c}} \leqslant \sqrt{3}$

 

 

 

Đặt biểu thức vế trái là $A$ . Áp dụng BĐT Cauchy Schwarz kết hợp với những BĐT sau:

 

1. $a+b+c\leq a^2+b^2+c^2=3$

 

2. $(a^2+b+c)(1+b+c)\geq (a+b+c)^2$ ta có

 

$A^2\leq (a+b+c)\left [ \sum \frac{a}{a^2+b+c} \right ]\leq (a+b+c)\left [ \sum \frac{a(1+b+c)}{(a+b+c)^2} \right ]$

 

$\Leftrightarrow A^2\leq \frac{a+b+c+2(ab+bc+ac)}{a+b+c}\leq \frac{(a+b+c)^2}{a+b+c}=a+b+c\leq 3$

 

$\rightarrow A\leq \sqrt{3}$ (đpcm)




#558615 CMR: $\sum \frac{a^{3}}{2a^{2...

Gửi bởi lahantaithe99 trong 10-05-2015 - 15:11



Cho a,b,c là các số thực. CMR: $\sum \frac{a^{3}}{2a^{2}-ab+3b^{2}} \geqslant \frac{a+b+c}{4}$

 

BĐT như thế này thì tự hiểu nó là nguyên dương!

 

Đặt biểu thức vế trái là $A$ thì áp dụng Cauchy Schwarz ta có

 

$A=\sum \frac{a^4}{2a^3-a^2b+3ab^2}\geq \frac{(a^2+b^2+c^2)^2}{2(a^3+b^3+c^3)-\sum a^2b+3\sum ab^2}\geq \frac{a+b+c}{4}$

 

$\Leftrightarrow 4(a^2+b^2+c^2)^2\geq (a+b+c)\left [ 2(a^3+b^3+c^3)-\sum a^2b+3\sum ab^2 \right ]$

 

$\Leftrightarrow 2(a^2+b^2+c^2)^2\geq \sum a^3b+5\sum ab^3$ $(*)$

 

Có BĐT phụ quen thuộc sau: $(a^2+b^2+c^2)^2\geq 3(a^3b+b^3c+c^3a)$ hoặc $(a^2+b^2+c^2)^2\geq 3(ab^3+bc^3+ca^3)$

 

Sử dụng BĐT trên ta có ngay $(*)$ nên ta có đpcm.

 

----------------------------------------------------------------------------

Chứng minh BĐT phụ

 

Áp dụng bất đẳng thức $(x+y+z)^2\geq 3(xy+yz+xz)$ thì đặt $(x,y,z)=(a^2+bc-ab,b^2+ac-bc,c^2+ab-ac)$ ta có ngay đpcm




#558599 ĐỀ THI OLYMPIC CHUYÊN KHOA HỌC TỰ NHIÊN NĂM 2015

Gửi bởi lahantaithe99 trong 10-05-2015 - 11:46

Ngày thi thứ hai

 

Câu IV. Cho $a,b\in\mathbb{Z}$, $n\in\mathbb{Z^+}$. CMR

$A=b^{n-1}a(a+b)(a+2b)...[a+(n-1)b]$ chia hết cho $n!$

 

Câu V. Cho tứ giác $ABCD$ nội tiếp $(O)$. Gọi $I,J$ lần lượt là tâm nội tiếp các tam giác $BAD,CAD$. Gọi $DI,AJ$ lần lượt cắt $(O)$ tại $S,T$. Đường thẳng $IJ$ cắt $AB,CD$ tại $M,N$.

a) Chứng minh rằng $SM,TN$ cắt nhau trên đường tròn $(O)$

b) Gọi đường tròn ngoại tiếp tam giác $ABN$ cắt $CD$ tại $P$ khác $N$. $(CDM)$ cắt $AB$ tại $Q$ khác $M$. Chứng minh rằng $PQ$ đi qua tâm nội tiếp hai tam giác $ABC$ và $DBC$

 

Câu VI. Cho $x,y,z>0$ và $xy+yz+xz=1$. CMR

$\frac{x}{\sqrt{yz}+\sqrt{3}}+\frac{y}{\sqrt{xz}+\sqrt{3}}+\frac{z}{\sqrt{xy}+\sqrt{3}}\leq \frac{1}{4\sqrt{3}xyz}$

 

---------------------------------------------------------------------------

P/s: Rớt rụng răng :))




#558433 ĐỀ THI OLYMPIC CHUYÊN KHOA HỌC TỰ NHIÊN NĂM 2015

Gửi bởi lahantaithe99 trong 09-05-2015 - 11:07

ĐỀ THI OLYMPIC CHUYÊN KHOA HỌC TỰ NHIÊN NĂM 2015

Môn thi: Toán

 

Ngày thi thứ nhất

 

Câu I: Tìm tất cả các số nguyên tố $p$ sao cho $3^p+4^p$ là số chính phương

 

Câu II. Cho tam giác $ABC$ tâm nội tiếp $(I)$ và $AI$ cắt $BC$ tại $D$. Một đường thẳng đi qua $A$ cắt đường tròn ngoại tiếp tam giác $IBC$ tại $P,Q$ sao cho $P$ nằm giữa $A,Q$.

a) CMR tích $DP.DQ$ không đổi khi $P,Q$ thay đổi

b) Giả sử đoạn thẳng $PQ$ cắt đoạn thẳng $BD$. Trên đoạn $DB$ lấy điểm $M$ sao cho $DM=DP$. Lấy $R$ đối xứng $M$ qua trung điểm $BC$. $(ADR)$ cắt $(IBC)$ tại $S,T$ . $ST$ cắt $BC$ tại $N$. CMR tam giác $DNQ$ cân.

 

Câu III. Hai bạn An và Bình chơi một trò chơi trên bảng vuông kích thước $3\times 2015$ ( $3$ hàng và $2015$ cột) . Hai người chơi lần lượt, An đi trước. Mỗi lần chơi, An đặt vào bảng một hình chữ nhật ngang $1\times 3$ và Bình đặt vào bảng một hình chữ nhật dọc $3\times 1$. Các hình chữ nhật được đặt vào không được chồng lên nhau. Ai đến lượt mình mà không đặt được hình chữ nhật là thua. Giả sử rằng cả hai bạn đều chơi rất giỏi. Hỏi ai có chiến thuật để chắc chắn dành được chiến thắng? 




#556167 $a^{2}+b^{2}+c^{2}+\frac{a+b+c...

Gửi bởi lahantaithe99 trong 25-04-2015 - 05:08

Cho các số dương a, b, c nhỏ hơn 1 thỏa mãn $(1-a)(1-b)(1-c)=abc$

Chứng minh: 

$a^{2}+b^{2}+c^{2}+\frac{a+b+c}{2}\geq \frac{3}{2}$

 

$(\frac{1-a}{a},\frac{1-b}{b},\frac{1-c}{c})=(x,y,z)\Rightarrow (a,b,c)=(\frac{1}{x+1},\frac{1}{y+1},\frac{1}{z+1})$ với $xyz=1$

 

Ta cần chứng minh $\sum (\frac{1}{x+1})^2+\frac{1}{2}\sum \frac{1}{x+1}\geq \frac{3}{2}$

 

Có 1 BĐT quen thuộc là $\frac{1}{(x+1)^2}+\frac{1}{(y+1)^2}\geq \frac{1}{xy+1}=\frac{z}{z+1}$

 

Theo nguyên lí Dirichle giả sử $(x-1)(y-1)\geq 0$ thì $x+y\leq xy+1$ nên

 

$\frac{1}{x+1}+\frac{1}{y+1}\geq\frac{4}{x+y+2}\geq\frac{4}{xy+3}=\frac{4z}{1+3z}$

 

Do đó cần chứng minh $\frac{1}{(z+1)^2}+\frac{z}{2(z+1)}+\frac{2z}{1+3z}\geq 1\Leftrightarrow \frac{z(z-1)^2}{(z+1)^2(1+3z)}\geq 0$ ( đúng)

 

Nên ta có đpcm

 




#550656 tìm nghiệm nguyên $2^a.3^b-5^x.7^y=1$

Gửi bởi lahantaithe99 trong 01-04-2015 - 02:20

tìm các bộ $(a,b,x,y)\in \mathbb{N}^*$ thỏa

$2^a.3^b-5^x.7^y=1$

 

Ta có $2^a3^b-1=5^x7^y$ nên $5|2^a3^b-1$ $(1)$ và $7|2^a3^b-1$ $(2)$

 

  Xét modun $4$ cho $a,b$ ta suy ra được rằng $a,b$ phải cùng tính chẵn lẻ mới thỏa mãn $(1)$

 

Xét modun $6$ cho $a,b$ ta suy ra $(a,b)$ có các dạng là

 

$(6k,6m), (6k+1,6m+4),(6k+2,6m+2), (6k+3,6m), (6k+4,6m+4), (6k+5,6m+2)$ 

 

để thỏa mãn $(2)$

 

Như vậy suy ra để thỏa mãn cả $(1)$ và $(2)$ thì $a,b$ phải cùng chẵn. Đặt $a=2c,b=2d$ ( $c,d\in\mathbb{N^*}$)

 

Phương trình trở thành $5^x7^y=(2^c3^d-1)(2^c3^d+1)$

 

Đến đây chắc là xét phương trình tích   :D




#547228 CMR $a^2+2b^2\vdots p\Leftrightarrow$ $a$ và...

Gửi bởi lahantaithe99 trong 14-03-2015 - 23:31

$1,$ cho $a,b$ nguyên và $p$ là số nguyên tố có dạng $8k+5(k\in \mathbb{N})$

CMR $a^2+2b^2\vdots p\Leftrightarrow \left\{\begin{matrix} a\vdots p\\b\vdots p \end{matrix}\right.$

 

NTP

Bài 1:

 

Giả sử $a,b$ không chia hết cho $p$ tức là $(a,p)=(b,p)=1$

 

Áp dụng định lý Fermat có $\left\{\begin{matrix} a^{p-1}= (a^4)^{2k+1}\equiv 1 (mod p) & \\ (4b^4)^{2k+1}\equiv 4^{2k+1} (mod p) & \end{matrix}\right.\Rightarrow 4^{2k+1}-1\equiv (a^2+2b^2)(...)\equiv 0 (mod p)$

 

Hay $2^{\frac{p-1}{2}}\equiv 1$ (mod $p$ ) $(1)$

 

Theo tiêu chuẩn Euler thì $2^{\frac{p-1}{2}}\equiv \left ( \frac{2}{p} \right )$ (mod $p$ )

 

Mà do $p$ là số nguyên tố có dạng $8k+5$ nên $\left ( \frac{2}{p} \right )=(-1)^{\frac{p^2-1}{8}}\equiv -1$ (mod $p$ ) $(2)$

 

Từ $(1)$ và $(2)$ ta thấy mâu thuẫn nên suy ra đpcm